LR Question Stem Recognition Flashcards

1
Q
  1. Which one of the following most accurately expresses the main conclusion drawn in the argument?
A

Main Point / Down

How well did you know this?
1
Not at all
2
3
4
5
Perfectly
2
Q
  1. Which one of the following is most strongly supported by the information above?
A

MSS / Down

How well did you know this?
1
Not at all
2
3
4
5
Perfectly
3
Q
  1. The claim that some painters are superior to others in the execution of their artistic visions plays which one of the following roles in the argument?
A

Argu Part / Down

How well did you know this?
1
Not at all
2
3
4
5
Perfectly
4
Q
  1. The economist’s statements, if true, most strongly support which one of the following?
A

MSS / Down

How well did you know this?
1
Not at all
2
3
4
5
Perfectly
5
Q
  1. Which one of the following is best supported by the information above?
A

MSS / Down

How well did you know this?
1
Not at all
2
3
4
5
Perfectly
6
Q
  1. Of the following, which one, if true, is the logically strongest counter that Cassie can make to Melvin’s argument?
A

Weaken / Up

How well did you know this?
1
Not at all
2
3
4
5
Perfectly
7
Q
  1. Which one of the following most accurately expresses the overall conclusion of the argument?
A

MP / Down

How well did you know this?
1
Not at all
2
3
4
5
Perfectly
8
Q
  1. Which one of the following most accurately expresses the main conclusion drawn in Leslie’s argument?
A

MP / Down

How well did you know this?
1
Not at all
2
3
4
5
Perfectly
9
Q
  1. Which one of the following statements is most supported by the information above?
A

MSS / Down

How well did you know this?
1
Not at all
2
3
4
5
Perfectly
10
Q
  1. Which one of the following most precisely describes the role played in the scientist’s argument by the statement that
A

Arg Part / Down

How well did you know this?
1
Not at all
2
3
4
5
Perfectly
11
Q
  1. Which one of the following, if true, most weakens the argument?
A

Weaken / Up

How well did you know this?
1
Not at all
2
3
4
5
Perfectly
12
Q
  1. If the shareholder’s statements are true, which one of the following is most strongly supported by them?
A

Most Strongly Supported / Down

How well did you know this?
1
Not at all
2
3
4
5
Perfectly
13
Q
  1. Which one of the following, if true, most seriously undermines the conclusion drawn in the argument above?
A

Weaken / Up

How well did you know this?
1
Not at all
2
3
4
5
Perfectly
14
Q
  1. Which one of the following would, if true, most weaken the psychologist’s argument?
A

Weaken / Up

How well did you know this?
1
Not at all
2
3
4
5
Perfectly
15
Q
  1. Which one of the following statements follows logically from the statements above?
A

Must Be True / Down

How well did you know this?
1
Not at all
2
3
4
5
Perfectly
16
Q
  1. Which one of the following can be properly inferred from the ecologist’s statements?
A

Must Be True / Down

How well did you know this?
1
Not at all
2
3
4
5
Perfectly
17
Q
  1. Which one of the following, if true, most weakens the argument?
A

Weaken / Up

How well did you know this?
1
Not at all
2
3
4
5
Perfectly
18
Q
  1. The lawyer’s conclusion follows logically if which one of the following is assumed?
A

Sufficient Assumption / Up

How well did you know this?
1
Not at all
2
3
4
5
Perfectly
19
Q
  1. Which one of the following, if true, most strengthens the argument?
A

Strengthen / Up

20
Q
  1. The conclusion of the psychiatrist’s argument is most strongly supported if which one of the following completes the argument?
A

Strengthen / Up

21
Q
  1. Which one of the following principles, if valid, most helps to justify the reasoning in the argument?
A

Pseudo Sufficient Assumption / Up

22
Q
  1. Which one of the following statements, if true, most weakens the reasoning above?
A

Weakening / Up

23
Q
  1. Which one of the following, if true, lends the most support to the psychologist’s conclusion?
A

Strengthening / Up

24
Q
  1. Which one of the following principles, if valid, most helps to justify the reasoning in the advice columnist’s argument?
A

Pseudo Sufficient Assumption / Up

25
Q
  1. Which one of the following, if assumed, allows the conclusion of the therapist’s argument to be properly inferred?
A

Sufficient Assumption / Up

26
Q
  1. The critic’s conclusion follows logically if which one of the following is assumed?
A

Sufficient Assumption / Up

27
Q
  1. Which one of the following statements, if true, most strengthens the argument?
A

Strengthen / Up

28
Q
  1. If the statements above are true, which one of the following is most strongly supported on the basis of them?
A

Most Strongly Supported / Down

29
Q
  1. Which one of the following, if true, adds the most support to the argument?
A

Strengthen / Up

30
Q
  1. Which one of the following principles, if valid, most helps to justify the real estate agent’s argumentation?
A

Pseudo Sufficient Assumption / Up

31
Q
  1. Which one of the following can be properly inferred from the information above?
A

Must Be True / Down

32
Q
  1. The information above provides the most support for which one of the following propositions?
A

Most Strongly Supported / Down

33
Q
  1. Which one of the following, if true, would most weaken the argument in the newspaper article?
A

Weaken / Up

34
Q
  1. The statements above, if true, most strongly support which one of the following?
A

Most Strongley Supported / Down

35
Q
  1. Which one of the following most accurately expresses the conclusion drawn in the botanist’s argument?
A

Main Point / Down

36
Q
  1. Which one of the following is most strongly supported by the information above?
A

Most Strongly Supported / Down

37
Q
  1. Which one of the following, if true, most weakens the argument above?
A

Weakening / Up

38
Q
  1. The argument’s conclusion is properly drawn if which one of the following is assumed?
A

Sufficient Assumption / Up

39
Q
  1. Which one of the following principles most helps to justify the mathematics teacher’s argument?
A

Pseudo Sufficient Assumption / Up

40
Q
  1. The conclusion of the argument follows logically if which one of the following is assumed?
A

Sufficient Assumption / Up

41
Q
  1. Which one of the following, if true, most strengthens the economist’s reasoning?
A

Strengthen / Up

42
Q
  1. Which one of the following, if true, would most strengthen the archaeologist’s reasoning?
A

Strengthen / Up

43
Q
  1. Which one of the following most accurately describes the role played in the farmer’s argument by the proposition that
A

Argument Part / Down

44
Q
  1. Which one of the following most accurately expresses the main conclusion of the gardener’s argument?
A

Main Point / Down

45
Q
  1. Which one of the following, if true, most weakens the argument above?
A

Weakening / Up

46
Q
  1. Which one of the following principles, if valid, most helps to justify Marian’s argument?
A

Pseudo Sufficient Assumption / Up